A health food store has launched a new pack of vitamins for children. The store records the number of packs ordered by a random sample of 100 online customers and the number of packs ordered by a random sample of 100 walk-in customers.

Two dot plots are shown one below the other. The title for the top dot plot is Online Customers and the title for the bottom plot is Walk In Customers. Below the line for each dot plot is written Number of Packs. There are markings from 1 to 5 on each line at intervals of one. For the top line there are 0, 0, 1, 4, 5 dots respectively for marks from 1 to 5. For the bottom line there are 3, 6, 1, 0, 0 dots respectively for marks from 1 to 5. On the right side, towards the middle of the two plots is shown a dot symbol equal to 10 customers.

Which inference can be drawn about the buying pattern of the online customers versus that of the walk-in customers?

Online customers buy an average of 4 packs compared with walk-in customers, who buy an average of 1 pack.
Online customers buy an average of 5 packs compared with walk-in customers, who buy an average of 2 packs.
Online customers buy an average of 4 packs compared with walk-in customers, who buy an average of 2 packs.
Online customers buy an average of 5 packs compared with walk-in customers, who buy an average of 1 pack.

A Health Food Store Has Launched A New Pack Of Vitamins For Children. The Store Records The Number Of

Answers

Answer 1

Answer:

The Answer is the second one

Step-by-step explanation:


Related Questions

A hockey puck strikes a wall at an angle of 30 degrees. the puck then travels away from the wall at the same angle.

Answers

Two or more angles whose sum is 180° are called supplementary angles. The measure of the ∠y is 120°.

What are supplementary angles?

Two or more angles whose sum is 180° are called supplementary angles. If a straight line is intersected by a line, then there are two angles form on each of the sides of the considered straight line. Those two-two angles are two pairs of supplementary angles. That means, that if supplementary angles are aligned adjacent to each other, their exterior sides will make a straight line.

Given the puck strikes the wall at an angle of 30°, it goes away at the same angle of 30°. Therefore, the measure of angle y can be found using the sum of the angle as a supplementary angle. Thus, we can write,

30° + ∠y + 30° = 180°

60° + ∠y = 180°

∠y = 180° - 60° = 120°

Hence, the measure of the ∠y is 120°.

Learn more about Supplementary Angles:

https://brainly.com/question/2882938

#SPJ1

An athlete completes one round of a circular track of diameter 200 m in 40 s what will be the distance covered and displacement at the end of 2 min 20s

Answers

Answer:

700π m ~ 2199 m

Step-by-step explanation:

diameter = d = 200 m

2 min 20s = 140 s

1 round / 40 s

140 = 40 * 3 + 20

so the athlete will cover 3 round and half

1 round = d * π

covered = c = 3 * 200 * π + 200/2 * π = 700 * π

The rectangular prism below has a base area of 44.3 units2 and a volume of 221.5
a
units. Find its height.

Answers

Given:

[tex]base \: area = 44.3 \: {units}^{2} [/tex]

[tex]volume = 221.5 \: {units}^{3} [/tex]

To find:

the height of the given rectangular prism.

Solution:

[tex]volume= base \: area \times height[/tex]

[tex]height = \frac{volume}{base \: area} [/tex]

[tex]height = \frac{221.5}{44.3} [/tex]

[tex]height = 5 \: units[/tex]

Therefore, the height of the given rectangular prism is 5 units.

The value of height of rectangular prism is,

⇒ h = 5 units

What is mean by Rectangle?

A rectangle is a two dimension figure with 4 sides, 4 corners and 4 right angles. The opposite sides of the rectangle are equal and parallel to each other.

Given that;

The rectangular prism below has a base area of 44.3 units² and a volume of 221.5 units³.

Since, The volume of rectangular prism is,

⇒ V = Base area x height

⇒ 221.5 = 44.3 × h

⇒ h = 221.5 / 44.3

⇒ h = 5 units

Thus, The value of height of rectangular prism is,

⇒ h = 5 units

Learn more about the rectangle visit:

https://brainly.com/question/2607596

#SPJ2

earth makes one full rotation on its axis in a day how long does it take for it to rotate 150 degrees​

Answers

Earth rotates one full rotation a day

360° in 3600s

So

It rotates 1° in

3600/360=10s

It rotates 150° in

150(10)1500s

How do I Factorise X^2 - 25

Answers

To Factorize Something:

 ⇒ means to put it into the simplest terms that when multiplied

     together transforms into the original form

Let's Solve:

  [tex]x^2-25=(x+5)(x-5)[/tex]

Let's Check:

  [tex](x+5)(x-5)=x*x+(-5)x+5x-25=x^2-5x+5x-25=x^2-25[/tex]

Answer: (x+5)(x-5)

Hope that helps!

Answer:

(x + 5)(x - 5)

Step-by-step explanation:

[tex]x^2-25[/tex]

[tex]=x^2-5^2[/tex]

[tex]x^2-5^2=\left(x+5\right)\left(x-5\right)[/tex]

[tex]=\left(x+5\right)\left(x-5\right)[/tex]

Calculate the distance between the pairs of coordinates, and classify them according to the distance between them.
(3, 4) and (2, 1)
(3, 7) and (5, 2)
(5, -2) and (3, 3)
(-2, 3) and (1, 2)
(-4, -2) and (-3, 1)
(4, -1) and (-1, 1)

Answers

Answer:

the first one is 3

second. is -5/2

third is 5/-2

fourth is -1/3

fifth is 3

sixth is 0/-5

Answer:

Square Root of 10: (3, 4) and (2,1)

(-4,-2) and (-3, 1)

(-2, 3) and (1,2)

Square Root of 29:

(3,7) and (5,2)

(5, -2) and (3,3)

(4,-1) and (-1,1)

Step-by-step explanation: Just took the test

f(x)=(x+3)^2 -8 how many real solutions?

Answers

Answer:

2 real solutions

Step-by-step explanation:

[tex]f(x)=(x+3)^2-8\\\\0=(x+3)^2-8\\\\8=(x+3)^2\\\\\pm\sqrt{8}=x+3\\\\x=-3\pm\sqrt{8}[/tex]

Therefore, the function set at [tex]f(x)=0[/tex] has two real solutions (x-intercepts).

You can also check the discriminant of the function by expanding it:

[tex](x+3)^2-8=x^2+6x+9-8=x^2+6x+1\\\\b^2-4ac=6^2-4(1)(1)=36-4=32 > 0[/tex]

Since it's greater than 0, there's two real solutions

-3(7+6t)
Can you tell me the answer

Answers

Answer: -21 + -18t

Step-by-step explanation:

-3(7+6t)

-3 x 7= -21

-3 x 6t= 18t

In which number is the value represented by the digit in the hundreds place 10 times
the value represented by the digit in the tens place?

4,771
3,389
3,244
1,437

Answers

Answer:

4,771

Step-by-step explanation:

In the number 4771, the hundreds digit (7) is 10 times the tens digit (7).

CL 1-146. Solve for the variable(s) in each triangle below. a. 15,20, and 15.​

Answers

The variable(s) in each triangle below 9, 12 and 16.

What is Area of Triangle?

The basic formula to find the area of a triangle is, area of triangle = 1/2 (b × h); where 'b' is the base and 'h' is the height of the triangle.

Let the base of the triangle be 15.

So, the area is:

Area= 1/2*b*h

       = 1/2*15*20

       = 150.

Let the base of the triangle be 25.

So, the area is:

Area= 1/2*b*h

  150= 1/2*25*h

       = 300/25

    h =12

Now, using Pythagoras Theorem

15²= h²+x²

15²=12²+x²

225-144= x²

x²= 81

x=±9

Now, x+y=25

            y=16

Thus , values are 9,12 and 16.

Learn more about Area of Triangle here:

https://brainly.com/question/27683633

#SPJ1

Give the general solution in degrees and radians. 9) 8cosØ = 4​

Answers

Hi Student!

The first step that we need to take before solving this problem is understand what we need to do and what information we are given to do that.  So first they are asking for us to determine the solution (or find theta) in both degrees and radians.  The information that is given to us is the equation [tex]8* cos (\theta) = 4[/tex].

After completing the initial step, the next step is to actually start solving the equation for the unknown which in this case is theta or the angle.

Divide both sides by 8

[tex]8* cos (\theta) = 4[/tex][tex]\frac{8* cos (\theta)}{8} = \frac{4}{8}[/tex][tex]cos (\theta) = \frac{1}{2}[/tex][tex]cos (\theta) = 0.5[/tex]

Multiply by the inverse of cos

[tex]cos^{-1}(cos (\theta)) = cos^{-1}(0.5)[/tex][tex]\theta = cos^{-1}(0.5)[/tex][tex]\theta = 60[/tex]

After solving for the unknown or theta, we were able to determine that the solution for our equation is 60 degrees.  Now we can move onto the final step which is to convert our answer to radians to complete the final part of the problem statement.

We can convert from degrees to radians by multiplying by π/180 which would get rid of degrees and give us radians.

Plug in the values

[tex]degrees*\frac{\pi }{180\ degrees}[/tex][tex]60\ degrees*\frac{\pi }{180\ degrees}[/tex]

Simplify the expression

[tex]\frac{\pi\ *\ 60\ degrees }{180\ degrees}[/tex][tex]\frac{\pi}{3}[/tex]

Therefore, our final answer is that our solution in degrees is 60 degrees and our solution in radians is π/3.

Help me on this please!

Answers

[tex]\qquad \textit{Amount for Exponential Growth} \\\\ A=P(1 + r)^t\qquad \begin{cases} A=\textit{accumulated amount}\\ P=\textit{initial amount}\dotfill &25\\ r=rate\to 300\%\to \frac{300}{100}\dotfill &3\\ t=\textit{elapsed time}\dotfill &x\\ \end{cases} \\\\\\ A=25(1 + 3)^{x}\implies A=25(4)^x~\hfill f(x)=25(4)^x[/tex]

4 is really 1 + 3, namely the growth rate is 300/100 or 300%.

so the quantity today, is 300% more than yesterday, so whatever it was yesterday plus three times that much, so the quantity is 4 times as large as yesterday's.

The two data sets shown below are almost identical. Which of the following
statements are true? (Select TWO)
Data Set 1: 32 41 51 63 66 71
Data Set 2: 32, 41, 51, 3 66, 71
The range of Data Set 2 is smaller than the range of Data Set 1.
The median of Data Set 2 is exactly 30 less than the median of Data Set 1.
The mean of Data Set 2 is exactly 10 less than the mean of Data Set 1.
The standard deviation of Data Set 2 is larger than the standard deviation of Data Set 1.

Answers

Based on the calculations, the two (2) statements which are true are:

The mean of Data Set 2 is exactly 10 less than the mean of Data Set 1.Option D.

How to calculate the range?

Mathematically, range can be calculated by using this formula;

Range = Highest number - Lowest number

Range 1 = 71 - 32

Range 1 = 39.

Range 2 = 71 - 3

Range 2 = 68.

Therefore, the range of Data Set 2 is not smaller than the range of Data Set 1.

How to calculate the mean?

Mathematically, the mean for these data sets would be calculated by using this formula:

Mean = [F(x)]/n

For the total number of Data Set 1, we have:

F(x) = 32 + 41 + 51 + 63 + 66 + 71

F(x) = 324.

Substituting the parameters into the formula, we have:

Mean = [F(x)]/n

Mean 1 = [324]/6

Mean 1 = 54.

For the total number of Data Set 2, we have:

F(x) = 32 + 41 + 51 + 3 + 66 + 71

F(x) = 324.

Substituting the parameters into the formula, we have:

Mean 2 = [264]/6

Mean 2 = 44.

Difference = Mean 1 - Mean 2

Difference = 54 - 44

Difference = 10.

Therefore, the mean of Data Set 2 is exactly 10 less than the mean of Data Set 1.

For the standard deviation of Data Set 1, we have:

SD₁ = √(1/n × ∑(xi - u₁)²)

SD₁ = √(1/5 × ∑(32 - 54)² + 1/5 × ∑(41 - 54)² + 1/5 × ∑(51 - 54)² + 1/5 × ∑(63 - 54)² + 1/5 × ∑(66 - 54)² + 1/5 × ∑(71 - 54)²)

SD₁ = 15.34.

For the standard deviation of Data Set 2, we have:

SD₂ = √(1/n × ∑(xi - u₂)²)

SD₂ = √(1/5 × ∑(32 - 44)² + 1/5 × ∑(41 - 44)² + 1/5 × ∑(51 - 44)² + 1/5 × ∑(3 - 44)² + 1/5 × ∑(66 - 44)² + 1/5 × ∑(71 - 44)²)

SD₂ = 24.88.

Therefore, the standard deviation of Data Set 2 is larger than the standard deviation of Data Set 1.

Read more on mean here: https://brainly.com/question/9550536

#SPJ1

PLease Help its a math problem !!! Which statement is true based on the table below?

Answers

Answer:

slope of f(x) is 3

Step-by-step explanation:

Slope of f(x) :

m = 4 - 1 / -2 + 3m = 3/1m = 3Correct

Y-intercept of g(x) :

when x = 0y-intercept is 1Wrong

g(x) is an exponential function ⇒ Wrongf(x) is a linear function ⇒ Wrong

The 1st option is correct.

A survey was given to a random sample of 45 residents of a town to determine
whether they support a new plan to raise taxes in order to increase education
spending. of those surveyed, 9 respondents said they were in favor of the plan. at the
95% confidence level, what is the margin of error for this survey expressed as a
proportion to the nearest thousandth?.

Answers

Using the z-distribution, it is found that the margin of error for the 95% confidence interval is of 0.117.

What is a confidence interval of proportions?

A confidence interval of proportions is given by:

[tex]\pi \pm z\sqrt{\frac{\pi(1-\pi)}{n}}[/tex]

In which:

[tex]\pi[/tex] is the sample proportion.z is the critical value.n is the sample size.

The margin of error is given by:

[tex]M = z\sqrt{\frac{\pi(1-\pi)}{n}}[/tex]

In this problem, we have a 95% confidence level, hence[tex]\alpha = 0.95[/tex], z is the value of Z that has a p-value of [tex]\frac{1+0.95}{2} = 0.975[/tex], so the critical value is z = 1.96.

The sample size and the estimate are given as follows:

[tex]n = 45, \pi = \frac{9}{45} = 0.2[/tex]

Hence the margin of error is given by:

[tex]M = z\sqrt{\frac{\pi(1-\pi)}{n}}[/tex]

[tex]M = 1.96\sqrt{\frac{0.2(0.8)}{45}}[/tex]

M = 0.117.

More can be learned about the z-distribution at https://brainly.com/question/25890103

#SPJ1

PLS ANSWER WITH EXPLANATION!!!!!!! 50 points and brainliest for best answer!!!!1
Theresa uses a unique box in the shape of a trapezoidal prism for her specialty candles. The area of the base of the box for the smaller candle is 125 cm2 and the box is 12 cm tall. The box used for the larger candle has a base whose area is 160% that of the smaller box, and it is 6 centimeters taller.
What percent of the volume of the smaller box is the volume of the larger box?
Answer:

Answers

Answer:

82cm

Step-by-step explanation:

The volume of the larger box is 240% that of the volume of the smaller box.

What is an equation?

An equation is an expression that shows the relationship between two or more numbers and variables

Volume of smaller box = area of base * height = 125 cm² * 12 cm = 1500 cm³

Volume of larger box = area of base * height = (1.6 * 125 cm²) * (12 + 6) cm = 3600 cm³

Ratio of volume = 3600/1500 = 2.4 = 240%

The volume of the larger box is 240% that of the volume of the smaller box.

Volume of the smaller box

Area of base ×Height125(12)1400cm³

Area of base of larger box

160% of 1251.6×125200cm²

Volume

200(6)1200cm³

Percentage

1400/1200×100116.67%

Lottie buys a pack of 50 cans of lemonade.
She pays £17 for the cans.

Lottie sell 32 of the cans for 50p each.
She sells the remaining cans for 20p each.

Work out Lottie's percentage profit.
Give your answer correct to three significant figures.​

Answers

Answer:

15.3%

In Order:

1) Lotte buys 50 cans for £17.

2) Sells 32 cans for 50 penny each.

3) Sells remaining for 20 penny each.

Solve:

Total cost for 32 cans:

price of each can × total cans

50 penny × 32

1600 penny

Remaining Cans : 50 - 32 = 18 cans

Total cost of this 18 cans:

20 penny × 18

360 penny

[tex]\boxed{\bf 100 \ penny \ (p) = 1 \ pound \ ( \sf \£ )}[/tex]

Total Gained : 1600 + 360 = 1960 pence ≈ £19.6

Profit : selling price - cost price : 19.6 - 17 = £2.6

Profit In percentage:

[tex]\sf \dfrac{(selling \ price - cost \ price)\ x \ 100}{cost \ price}[/tex]

[tex]\rightarrow \sf \dfrac{2.6\ x \ 100}{17}[/tex]

[tex]\rightarrow \sf 15.294 \%[/tex]

[tex]\rightarrow \sf 15.3 \% \ \ (rounded \ to \ nearest \ three \ significant \ figure)[/tex]

Lottie's percentage profit based on the provided information is 15.29%

How can the percent be found?

A value or ratio that may be stated as a fraction of 100 is referred to as a percentage in mathematics. If we need to calculate a percentage of a number, we should divide it by its entirety and then multiply it by 100. The proportion therefore refers to a component per hundred. Per cent refers to one hundred percent.

The cost will be  £17 according to the question

Total  amount generated are;

32 cans * £0.50=£16

18 cans * £0.20=£3.60

Then the total will be £16 + £3.60 = £19.60

Profit =  amount generated- cost

=£19.60 - £17

= £2.60

The percentage profit:

[tex]= \frac{2.60}{17} * 100 \\\\=[/tex]

15.29%

Learn more about percentage at

https://brainly.com/question/24877689

#SPJ3

how many quarts are in 2 gallons?

Answers

Answer:

8 quarts

Step-by-step explanation:

There are 4 quarts in 1 gallon

So in 2 gallons there would be 4 × 2 = 8 quarts

Its 2 Lquid gallons = 8 Liquid quart

I don't know pls help

Answers

Answer:

125

Step-by-step explanation:

JIK is congruent to MIL so they both equal 55. 360-55=305-55=250. now MIJ and LIK are gonna be equal so you can divide 250 by 2 which gives you 125. if you add them up it equals 360, a full circle! pls give me brainliest!

PLEASE HELP!
Before you answer:
-No spam (Will be reported)
-No incorrect answers (Will be reported)
-Correct answers and explanations will be mark as BRAINLIEST​​

Answers

Answer:

Step-by-step explanation:

Sphere:

1) a) r =  7 cm

 [tex]\sf \boxed{\text{Volume of sphere=$\dfrac{4}{3}\pi r^3$}}[/tex]

                             [tex]\sf =\dfrac{4}{3}*3.14*7*7*7\\\\= 1436 \ cm^3[/tex]

 [tex]\sf \boxed{\text{\bf Surface area of sphere = $4\pi r^2$}}[/tex]

                                           = 4*3.14 * 7 * 7

                                           = 615.44 cm²

b) r= 8.4 cm

     [tex]Volume = \dfrac{4}{3}\pi r^3[/tex]

                  [tex]\sf =\dfrac{4}{3}*\dfrac{22}{7}*8.4*8.4*8.4\\\\= 2483.7 \ cm^3[/tex]

   Surface area = 4*3.14*8.4*8.4

                         = 887.04 cm²

Hemisphere:

      a) r = 6 cm

     [tex]\sf \boxed{\text{\bf Volume of hemisphere =$\dfrac{2}{3} \pi r^3$}}[/tex]

                                                [tex]\sf =\dfrac{2}{3}*3.14*6*6*6\\\\= 452.16 \ cm^3[/tex]

        [tex]\sf \boxed{\text{\bf Surface area of hemisphere=3 \pi r^2}}[/tex][tex]\sf \boxed{\text{\bf Surface area of hemisphere = $3 \pi r^2$}}[/tex]

                                                            = 3 * 3.14 * 6 * 6

                                                             = 339.12 cm²

                                                           

                       

A toy store is building a display with two rectangular prisms. The base of each prism is a square with an edge length of 6 inches. One prism is 10 inches tall and the other is twice as tall. The two prisms are stacked on top of each other. What is the total volume of the display?

Answers

If the two prisms are stacked on top of each other then the height of the prism is 30 inches. Then the volume of the prism will be 1080 in³.

What is a rectangular prism?

A rectangular prism is a closed solid that has two parallel rectangular bases connected by a rectangle surface.

A toy store is building a display with two rectangular prisms.

The base of each prism is a square with an edge length of 6 inches.

One prism is 10 inches tall and the other is twice as tall.

The two prisms are stacked on top of each other.

Then the total volume of the display will be

The volume of the prism will be

Volume = Base area × Height

Then the height of the second prism will be

h₂ = 2 × 10 = 20 inches

Then the volume of the prism will be

Volume = 6 × 6 × 10 + 6 × 6 × 20

Volume = 360 + 720

Volume = 1080 cubic inches

More about the rectangular prism link is given below.

https://brainly.com/question/12649592

#SPJ1

Which statements are true about a histogram with one-minute increments representing the data? select three options. a histogram will show that the mean time is approximately equal to the median time of 7.5 minutes. the histogram will have a shape that is left-skewed. the histogram will show that the mean time is greater than the median time of 7.4 minutes. the shape of the histogram can be approximated with a normal curve. the histogram will show that most of the data is centered between 6 minutes and 9 minutes.

Answers

If a histogram has one-minute increments representing the data, we will see that:

A histogram will show that the mean time is approximately equal to the median time of 7.5 minutesThe shape of the histogram can be approximated with a normal curve.The histogram will show that most of the data is centered between 6 minutes and 9 minutes.

What will the histogram show?

If there are is a histogram which has one-minute increments to represent the data, the shape will be that of a normal curve thanks to most data being  between the 6 and 9 minute mark.

The fact that the histogram will have a normal curve means that the mean will be approximately equal to the median because the mean and median are usually equal for normal distributions.

Find out more on normal distributions at https://brainly.com/question/23418254.

#SPJ4

Answer:

If a histogram has one-minute increments representing the data, we will see that:

A histogram will show that the mean time is approximately equal to the median time of 7.5 minutes

The shape of the histogram can be approximated with a normal curve.

The histogram will show that most of the data is centered between 6 minutes and 9 minutes.

Step-by-step explanation:

WILL MARK BRAINIEST

The average of the first three numbers were was six the average of the next 7 numbers was 19 what was the overall average of the 10 numbers

Answers

Answer:

3 1 4 7 9 3 8

Step-by-step explanation:

This net will be folded to form a prism.

A net of a rectangular prism. From top to bottom, the rectangles are labeled B, D, E, F. The left side is labeled A and the right side is labeled C.

Which pairs of faces will be opposite one another when the prism is created? Check all that apply.
D and F
A and C
A and B
B and F
B and E
B and C
E and F
A and F
Mark this and return

Answers

The opposite faces are D and F, A and C, and B and E. Then the correct options are A, B, and E.

What is Geometry?

It deals with the size of geometry, region, and density of the different forms both 2D and 3D.

This net will be folded to form a prism.

A net of a rectangular prism.

From top to bottom, the rectangles are labeled B, D, E, and F.

The left side is labeled A and the right side is labeled C.

Then the pairs of faces will be opposite one another when the prism is created will be

BD, DE, EF, FB, AD, DC, CF, FA, AB, BC, BE, and EA all are the adjacent side faces.

AC, BE, and DF are the opposite faces.

Then the correct options are A, B, and E.

More about the geometry link is given below.

https://brainly.com/question/7558603

#SPJ1

The area of the triangle is

Answers

Answer:

3units squared

Step-by-step explanation:

1/2 the base times the height

1/2 * 2 * 3 = 3

HELPPPP HURRY!!!!!!!!!
Describe one way you could graph the equation: y = 4x + 1,?

Answers

Answer:

Start at (0,1) and then go up 4 spaces over 1 to the left and repeat going up 4 over 1. To complete the graph by getting it to go behind the y intercept of (0,1) you go from the y intercept down 4 over 1 to the right and repeat going down 4 over 1.

For plotting a line, we need to plot points [tex](0,1)[/tex] and [tex](\frac{-1}{4},0)[/tex] on the cartesian plane and join them to form a line.

What is plotting a line?

For plotting a line, we only need two points to graph a line. With that idea, we take advantage of the two “convenient” points which can easily be solved algebraically from the equation of the line; namely, the x-intercept (x,0) and y-intercept (0,y).

[tex]y = 4x + 1\\\\Put\ x = 0 \\y = 1\\we\ get\ (0,1)\ as\ a\ point\ lying\ on\ the\ line. \\\\Put\ y = 0 \\x = \frac{-1}{4} \\we\ get\ (\frac{-1}{4} ,0)\ as\ a\ point\ lying\ on\ the\ line. \\[/tex]

Learn more about plotting a line here

https://brainly.com/question/16321364

#SPJ2

HELPHELEHEPELEHELPEGELPE

Answers

Answer: A= 68 square centimeters

Step-by-step explanation:

A= [tex]B^{1}[/tex] +[tex]B^{2}[/tex]/2 x h

6+11=17

17/2=8.5

8.5 x 8= 68

A lock has a three number code made up of 17 numbers if none of the numbers are allowed to repeat how many different ways can you choose three different numbers in order for unit code?

Answers

Answer:

680

Step-by-step explanation:

Use the binomial coefficient where you choose [tex]k=3[/tex] numbers out of [tex]n=17[/tex] possible numbers and find the total amount of combinations since order does not matter:

[tex]\displaystyle \binom{n}{k}=\frac{n!}{k!(n-k)!}\\ \\\binom{17}{3}=\frac{17!}{3!(17-3)!}\\\\\binom{17}{3}=\frac{17!}{3!(14)!}\\\\\binom{17}{3}=\frac{17*16*15}{3*2*1}\\\\\binom{17}{3}=680[/tex]

Thus, you can make 680 three-non-repeating-number codes

i will give you 100 points HELp

Answers

Answer:

CD ≈ 26.0 cm

Step-by-step explanation:

using the sine ratio in right triangle ABD

sin35° = [tex]\frac{opposite}{hypotenuse}[/tex] = [tex]\frac{AB}{BD}[/tex] = [tex]\frac{12}{BD}[/tex] ( multiply both sides by BD )

BD × sin35° = 12 ( divide both sides by sin35° )

BD = [tex]\frac{12}{sin35}[/tex] ≈ 20.92 cm

using the sine rule in Δ BCD

[tex]\frac{BD}{sinC}[/tex] = [tex]\frac{CD}{sinB}[/tex] , that is

[tex]\frac{20.92}{sin52}[/tex] = [tex]\frac{CD}{sin102}[/tex] ( cross- multiply )

CD × sin52° = 20.92 × sin102° ( divide both sides by sin52° )

CD = [tex]\frac{20.92sin102}{sin52}[/tex] ≈ 26.0 cm ( to 3 significant figures )

Answer:

CD = 26.0 cm (3 sf)

Step-by-step explanation:

Sine Rule for side lengths

[tex]\sf \dfrac{a}{\sin A}=\dfrac{b}{\sin B}=\dfrac{c}{\sin C}[/tex]

(where A, B and C are the angles and a, b and c are the sides opposite the angles)

Find length BD:

[tex]\implies \sf \dfrac{AB}{\sin ADB}=\dfrac{BD}{\sin BAD}[/tex]

[tex]\implies \sf \dfrac{12}{\sin 35^{\circ}}=\dfrac{BD}{\sin 90^{\circ}}[/tex]

[tex]\implies \sf BD=\dfrac{12\sin 90^{\circ}}{\sin 35^{\circ}}[/tex]

[tex]\implies \sf BD=20.92136155...cm[/tex]

Find length CD:

[tex]\implies \sf \dfrac{BD}{\sin BCD}=\dfrac{CD}{\sin DBC}[/tex]

[tex]\implies \sf \dfrac{20.921...}{\sin 52^{\circ}}=\dfrac{CD}{\sin 102^{\circ}}[/tex]

[tex]\implies \sf CD=\dfrac{20.921...\sin 102^{\circ}}{\sin 52^{\circ}}[/tex]

[tex]\implies \sf CD=25.96941667...cm[/tex]

Therefore, CD = 26.0 cm (3 sf)

can someone please tell me how many cups is 7/2 pints??

Answers

Answer:

8

Step-by-step explanation:

Other Questions
Please help I need help thank you so much Easy Middle school math not high school. I have been absent for the past 2 days at school and they were talking about range, mode, median, quartile, and I think that is it. It would really help if you guys can help me do all of this work :) The pictures are the ones that I have been stuck on. Will give brainlist mobob LOL What is the most equitable tax policy and why? Which of the following statements is true about the number 30/2?A. It is contained in the set of natural numbers, whole numbers, integers, and rational numbers, but it is not contained in the set of irrational numbers.B. It is contained in the set of irrational numbers and rational numbers, but it is not contained in the set of natural numbers or whole numbers.C. It is contained in the set of rational numbers, but it is not contained in the set of natural numbers, whole numbers, integers, or irrational numbers.D. It is contained in the set of whole numbers and rational numbers, but it is not contained in the set of irrational numbers or integers. The "brightness" or tallness of an electromagnetic wave is its:A) AmplitudeB) WavelengthC) Frequency Which factor is measured to determine the winner of the shot-put competition?A. TimeB. SpeedC. HeightD. Distance A client has thoughts that they cannot exercise on their own, that they look silly doing it, and that other people are watching them. they believe these thoughts to be true. what is occurring I need help with these two trigonometry problems.Find each measurement indicated. Round your answers to the nearest tenth. Hey here are some photos i took for my photography class! What do you think? He has us do themes and the themes i do are like stuff involving nature so thats why there are alot of flowers (part 1) As a restriction to most flextime programs, most employers require flextime employees to _________. a. take only two 10 minute breaks per day b. work more than 45 hours a week c. be at work during peak hours d. none of the above is typical A sample of argon has a volume of 0.43 ml at 299k. at what temperature in degrees celsius will it have a volume of 1 ml. What statement describes an electrolyte? In a class of 25 students, we want to distribute their exam paper results.What is the probability that only 2 students DO NOT get their exam paper. (23 get theirs and 2 are swapped). "Maybe hunger is not pretty in a woman. Maybe a ferocious appetite is unbecoming. But, no, those are only lies we've been told."what does this quote mean? What exercises can help reduce stress? PLEASE HELP ME!!!!!!!!!!!!! Passage 1 Do you know how Maryland and the city of Baltimore got their names? On March 25, Marylanders celebrate Maryland Day to honor the arrival of the first colonists to the land King Charles I of England chartered to Lord Baltimore in 1634. He named the land after the king's wife, Henrietta Maria, or Mary. Lord Baltimore had almost complete control over the colony as long as he paid the king a share of all the gold or silver discovered on the land.Passage 2 The Atlantic Coastal Plain covers about one-half of Maryland's land area, yielding to the Piedmont Plateau at a fall line running from the northern tip of the District of Columbia through Baltimore and to near the northeastern corner of the state. The Catoctin ridgeline in the west forms the gateway to the Appalachians.How are the two passages alike? A. Both passages are about American cities. B. Both passages are about natural land features. C. Both passages are about history. D. Both passages are about Maryland. "l will help you?" He said that,she________help him.a.wouldb.willc.hasd.had Given: DFE is isosceles with base FE; FB EC.Prove: DFB DECTriangle D F E is shown. 2 lines drawn down from point D to points C and B on the base of the triangle to form 3 triangles.Complete the missing parts of the paragraph proof.We know that triangle DFE is isosceles with base FE and that segment FB is congruent to segment EC because . Segment DF is congruent to segment by the definition of isosceles triangle. Since these segments are congruent, the base angles, angles , are congruent by the isosceles triangle theorem. Therefore, triangles are congruent by SAS. A firm in a perfectly competitive market: a.must reduce its price if it wants to sell a larger quantity. b.must be large relative to the total market. c.can exert a major influence on the market price. d.must take the price that is determined in the market.